Further explanation needed
Hello- I had a difficult time understanding this question. Can somebody please break this down...
@MichaelaJ on February 6 at 09:14PM
  • November 2018 LSAT
  • SEC4
  • Q14
3
Replies
Still need additional clarification
Hello- I am still not quite understanding why B is correct and D is incorrect. If we are looki...
@MichaelaJ on February 6 at 09:07PM
  • November 2018 LSAT
  • SEC1
  • Q12
2
Replies
C vs D
Hello Can you please explain what C & D are saying and how I can translate both of these to sc...
@MichaelaJ on February 6 at 08:57PM
  • November 2018 LSAT
  • SEC1
  • Q19
3
Replies
Incorrect Answer explanation
I understand the correct answer, but I was hoping someone could help explain what the explanation...
mbshapir on February 6 at 01:41PM
  • December 2017 LSAT
  • SEC3
  • Q24
1
Reply
Can someone please explain this correct answer?
Can someone please explain this correct answer? Thanks
jingjingxiao11111@gmail.com on February 6 at 04:16AM
  • December 1998 LSAT
  • SEC4
  • Q20
1
Reply
Can someone explain why C is correct?
I guess C but I dont understand why
capoleway@gmail.com on February 6 at 04:13AM
  • December 1998 LSAT
  • SEC4
  • Q15
1
Reply
How can I find the correct anwser?
From my understanding closed systems refer to her to a system in which she is unable to be innova...
_Hannah26 on February 6 at 03:36AM
  • June 1991 LSAT
  • SEC1
  • Q4
1
Reply
How to solve this question?
Can someone explain how to work through this question? I'm struggling to visualize even with the ...
Vennela-Vellanki on February 4 at 04:34PM
  • December 2016 LSAT
  • SEC3
  • Q19
1
Reply
Please help with the setup
Hello, not sure what type of game this is
LSATjourney on February 4 at 04:33PM
  • December 2016 LSAT
  • SEC3
  • Q19
1
Reply
Why is the answer A and why is B wrong?
Can I have help with running through this questions strategy?
Vennela-Vellanki on February 4 at 04:22PM
  • December 2016 LSAT
  • SEC3
  • Q13
1
Reply
Missing a rule?
Thanks
MWins on February 4 at 04:16PM
  • December 2016 LSAT
  • SEC3
  • Q4
1
Reply
B vs E
Hello I toggled between answer choices B & E ultimately choosing B. Can you please explain the...
@MichaelaJ on January 29 at 03:04PM
  • September 2016 LSAT
  • SEC2
  • Q22
1
Reply
What about h-s-m/v-m/v-t-l
If we used h-s-m/v-m/v-t-l H would be before T and still meet the m&v-T. So couldn't A be possible?
Jasmin1 on January 26 at 08:38PM
  • June 2016 LSAT
  • SEC2
  • Q14
1
Reply
Can you please explain?
I narrowed this down to A and B, but couldn't figure out how to get all the conditions to qualify...
Stefaniggorman on January 26 at 08:35PM
  • June 2016 LSAT
  • SEC2
  • Q8
1
Reply
half lists
what is the best way to go about half list questions like this without taking up too much time?
kassidee on January 26 at 08:27PM
  • October 2015 LSAT
  • SEC3
  • Q14
1
Reply
I do not see how C is correct.
It seems to me that Bernard is saying the exact opposite. Could someone please explain this to me?
MelissaEngelking on January 25 at 08:53PM
  • August 2020 LSAT
  • SEC3
  • Q3
1
Reply
why b?
Where in the passage can we find B. I thought it was in the start of the 2nd paragraph where sedi...
Elizabeth25 on January 25 at 08:52PM
  • October 2002 LSAT
  • SEC3
  • Q4
1
Reply
Answer choice D
In answer choice D, why can't M be in the first slot, Industry be in the second slot and M be in ...
amarachicynthia on January 25 at 08:49PM
  • June 2015 LSAT
  • SEC4
  • Q23
1
Reply
Set Up
For the set up, I used 1/3/5 along the base. Got all of the answers correct though it likely too...
bb042745 on January 25 at 08:34PM
  • June 2015 LSAT
  • SEC4
  • Q1
1
Reply
Distributions
How can you have a 1-2-2 scenario when the first two options are described as being solid or wove...
NS1 on January 25 at 08:28PM
  • December 2014 LSAT
  • SEC2
  • Q11
1
Reply